3
$\begingroup$

I am working on an optimization problem which I am stuck on towards the end.

Essentially, I have two probability density functions in $\mathbb{R}^2$, call them $q(x,y)$ and $p(x,y)$, now I define the objective functional to be:

$C(p,q) = \int p(x,y) \ln\left[\frac{p(x,y)}{q(x,y)}\right] dxdy \cdots (1)$

Now assume we already know what $q(x,y)$ is, ie, it is pre-determined, so my aim is to minimise $(1)$ with respect to the unknown density $p(x,y)$ subject to the following constraints:

$\int_{-\infty}^{\infty} \int_{X^x_d}^{\infty} p(x,y) dx dy = PoD^x$

$\int_{-\infty}^{\infty} \int_{X^y_d}^{\infty} p(x,y) dy dx = PoD^y$

$\int_{-\infty}^{\infty} \int_{-\infty}^{\infty} p(x,y) dx dy = 1$

$p(x,y) \ge 0 \ \forall (x,y) \in \mathbb{R}^2$

Now if we define:

$ I_{[X^x_d, \infty)} = \begin{cases} 1 & \text{if} x \ge X^x_d \\ 0 & \text{if} x < X^x_d \end{cases}$

$ I_{[X^y_d, \infty)} = \begin{cases} 1 & \text{if} y \ge X^y_d \\ 0 & \text{if} y < X^y_d \end{cases}$

Then our constraints become:

$\int \int p(x,y) I_{[X^x_d, \infty)} dx dy = PoD^x \cdots (2)$

$\int \int p(x,y) I_{[X^y_d, \infty)} dy dx = PoD^y \cdots (3)$

$\int \int p(x,y) dx dy = 1 \cdots (4)$

So I set up the Lagrangian as follows:

$L(p,q) = \int \int p(x,y) \ln[p(x,y)] dxdy - \int \int p(x,y) \ln[q(x,y)] dxdy + \lambda_1 \left[ \int \int p(x,y) I_{[X^x_d, \infty)} dx dy - PoD^x\right] + \lambda_2 \left[\int \int p(x,y) I_{[X^y_d, \infty)} dydx - PoD^y \right] + \mu \left[\int \int p(x,y) dxdy -1\right]$

which can further be simplified:

$L(p,q) = \int \int p(x,y)\left[\ln[p(x,y)] - \ln[q(x,y)]\right] dxdy + \int \int p(x,y) \left[\lambda_1 I_{[X^x_d, \infty)} + \lambda_2 I_{[X^y_d, \infty)} + \mu \right] dxdy - \lambda_1 PoD^x - \lambda_2 PoD^y - \mu$

Now to solve this for $p(x,y)$ I was told to use the calculus of variations and the optimal solution would be:

$\widehat{p(x,y)} = q(x,y) \exp\{-\left[1+\hat{\mu} + \left(\hat{\lambda_1} I_{[X^x_d, \infty)} \right) + \left(\hat{\lambda_2} I_{[X^y_d, \infty)} \right) \right] \}$

Now I am not very familiar with calculus of variations, I only really know how to optimize functionals with respect to algebraic constraints, however I am not sure what to do when the constraints are (multiple) integrals, any help would be appreciated :)

$\endgroup$
2
  • $\begingroup$ I'm not an expert in probability theory and statistics, so forgive me if my question is misguided. Already in your first equation, what happens if $q = 0$ but $p\neq 0$ on a set of positive measure? $\endgroup$ Jul 23, 2013 at 3:12
  • $\begingroup$ @TheoJohnson-Freyd: In such a case $C(p,q)$ is defined to be $\infty$. $\endgroup$
    – Ashok
    Nov 11, 2014 at 11:29

1 Answer 1

2
$\begingroup$

Just differentiate $L(p,q)$ with respect to $p(x,y)$, assuming regularities, to get

$$\frac{\partial}{\partial p(x,y)}L(p,q)=1+\ln p(x,y)-\ln q(x,y)+\lambda_1 I_{[X^x_d, \infty)} + \lambda_2 I_{[X^y_d, \infty)}+\mu$$ and equate to zero (the necessary condition in Lagrange multiplier method).

Now using the constraint that $\int \int p(x,y) dx dy = 1$, you see the optimising $p$ is given by $$p(x,y) = c\cdot q(x,y) \exp{\left(-(\lambda_1 I_{[X^x_d, \infty)}+\lambda_2 I_{[X^y_d, \infty)})\right)}$$

where $c$ is the normalising constant.

To confirm the above is indeed the solution, (if you are exposed to measure theory) you can have a look into the nice Ann. Probab. paper (Theorem 3.1) by I.Csiszar http://projecteuclid.org/DPubS?service=UI&version=1.0&verb=Display&handle=euclid.aop/1176996454.

$\endgroup$
1
  • $\begingroup$ $C(p,q)$ is a convex functional with respect to function $p$, so the stationary solution can be shown to be the unique minimal point. $\endgroup$
    – Hans
    Aug 27, 2014 at 16:20

Your Answer

By clicking “Post Your Answer”, you agree to our terms of service and acknowledge you have read our privacy policy.

Not the answer you're looking for? Browse other questions tagged or ask your own question.